K
Khách

Hãy nhập câu hỏi của bạn vào đây, nếu là tài khoản VIP, bạn sẽ được ưu tiên trả lời.

14 tháng 7 2017

quy đồng là ra

AH
Akai Haruma
Giáo viên
2 tháng 3 2017

Bài 3)

BĐT cần chứng minh tương đương với:

\(\left ( \frac{a}{a+b} \right )^2+\left ( \frac{b}{b+c} \right )^2+\left ( \frac{c}{c+a} \right )^2\geq \frac{1}{2}\left ( 3-\frac{a}{a+b}-\frac{b}{b+c}-\frac{c}{c+a} \right )\)

Để cho gọn, đặt \((x,y,z)=\left (\frac{b}{a},\frac{c}{b},\frac{a}{c}\right)\) \(\Rightarrow xyz=1\).

BĐT được viết lại như sau:

\(A=2\left [ \frac{1}{(x+1)^2}+\frac{1}{(y+1)^2}+\frac{1}{(z+1)^2} \right ]+\frac{1}{x+1}+\frac{1}{y+1}+\frac{1}{z+1}\geq 3\) \((\star)\)

Ta nhớ đến hai bổ đề khá quen thuộc sau:

Bổ đề 1: Với \(a,b>0\) thì \(\frac{1}{(a+1)^2}+\frac{1}{(b+1)^2}\geq \frac{1}{ab+1}\)

Cách CM rất đơn giản, Cauchy - Schwarz:

\((a+1)^2\leq (a+b)(a+\frac{1}{b})\Rightarrow \frac{1}{(a+1)^2}\geq \frac{b}{(a+b)(ab+1)}\)

Tương tự với biểu thức còn lại và cộng vào thu được đpcm

Bổ đề 2: Với \(x,y>0,xy\geq 1\) thì \(\frac{1}{x^2+1}+\frac{1}{y^2+1}\geq \frac{2}{xy+1}\)

Cách CM: Quy đồng ta có đpcm.

Do tính hoán vị nên không mất tổng quát giả sử \(z=\min (x,y,z)\)

\(\Rightarrow xy\geq 1\). Áp dụng hai bổ đề trên:

\(A\geq 2\left [ \frac{1}{xy+1}+\frac{1}{(z+1)^2} \right ]+\frac{2}{\sqrt{xy}+1}+\frac{1}{z+1}=2\left [ \frac{z}{z+1}+\frac{1}{(z+1)^2} \right ]+\frac{2\sqrt{z}}{\sqrt{z}+1}+\frac{1}{z+1}\)

\(\Leftrightarrow A\geq \frac{2(z^2+z+1)}{(z+1)^2}+\frac{1}{z+1}+2-\frac{2}{\sqrt{z}+1}\geq 3\)

\(\Leftrightarrow 2\left [ \frac{z^2+z+1}{(z+1)^2}-\frac{3}{4} \right ]+\frac{1}{z+1}-\frac{1}{2}-\left ( \frac{2}{\sqrt{z}+1}-1 \right )\geq 0\)

\(\Leftrightarrow \frac{(z-1)^2}{2(z+1)^2}-\frac{z-1}{2(z+1)}+\frac{z-1}{(\sqrt{z}+1)^2}\geq 0\Leftrightarrow (z-1)\left [ \frac{1}{(\sqrt{z}+1)^2}-\frac{1}{(z+1)^2} \right ]\geq 0\)

\(\Leftrightarrow \frac{\sqrt{z}(\sqrt{z}-1)^2(\sqrt{z}+1)(z+\sqrt{z}+2)}{(\sqrt{z}+1)^2(z+1)^2}\geq 0\) ( luôn đúng với mọi \(z>0\) )

Do đó \((\star)\) được cm. Bài toán hoàn tất.

Dấu bằng xảy ra khi \(a=b=c\)

P/s: Nghỉ tuyển lâu rồi giờ mới gặp mấy bài BĐT phải động não. Khuya rồi nên xin phép làm bài 3 trước. Hai bài kia xin khiếu. Nếu làm đc chắc tối mai sẽ post.

2 tháng 3 2017

Bài 1:

Cho \(a=b=c=\dfrac{1}{\sqrt{3}}\). Khi đó \(M=\sqrt{3}-2\)

Ta sẽ chứng minh nó là giá trị nhỏ nhất

Thật vậy, đặt c là giá trị nhỏ nhất của a,b,c. Khi đó, ta cần chứng minh

\(\frac{a^2}{b}+\frac{b^2}{c}+\frac{c^2}{a}-\frac{2(a^2+b^2+c^2)}{\sqrt{ab+ac+bc}}\geq(\sqrt3-2)\sqrt{ab+ac+bc}\)

\(\Leftrightarrow\sqrt{ab+ac+bc}\left(\frac{a^2}{b}+\frac{b^2}{c}+\frac{c^2}{a}-\sqrt{3(ab+ac+bc)}\right)\geq2(a^2+b^2+c^2-ab-ac-bc)\)

\(\Leftrightarrow\frac{a^2}{b}+\frac{b^2}{a}-a-b+\frac{b^2}{c}+\frac{c^2}{a}-\frac{b^2}{a}-c+a+b+c-\sqrt{3(ab+ac+bc)}\geq\)

\(\geq2((a-b)^2+(c-a)(c-b))\)

\(\Leftrightarrow(a-b)^2\left(\frac{1}{a}+\frac{1}{b}-2\right)+(c-a)(c-b)\left(\frac{1}{a}+\frac{b}{ac}-2\right)+a+b+c-\sqrt{3(ab+ac+bc)}\geq0\)

Đúng bởi \(\frac{1}{a}+\frac{1}{b}-2>0;\frac{1}{a}+\frac{b}{ac}-2\geq\frac{1}{a}+\frac{1}{a}-2>0\)

\(a+b+c-\sqrt{3(ab+ac+bc)}=\frac{(a-b)^2+(c-a)(c-b)}{a+b+c+\sqrt{3(ab+ac+bc)}}\geq0\)

BĐT đã được c/m. Vậy \(M_{Min}=\sqrt{3}-2\Leftrightarrow a=b=c=\dfrac{1}{\sqrt{3}}\)

P/s: Nhìn qua thấy ngon mà làm mới thấy thật sự là "choáng"

Giúp mình với! Mình đang cần gấp. Các bạn làm được bài nào thì giúp đỡ mình nhé! Cảm ơn!Bài 1: Cho các số thực dương a,b,c. Chứng minh rằng:\(\frac{a^2}{\sqrt{\left(2a^2+b^2\right)\left(2a^2+c^2\right)}}+\frac{b^2}{\sqrt{\left(2b^2+c^2\right)\left(2b^2+a^2\right)}}+\frac{c^2}{\sqrt{\left(2c^2+a^2\right)\left(2c^2+b^2\right)}}\le1\).Bài 2: Cho các số thực dương a,b,c,d. Chứng minh...
Đọc tiếp

Giúp mình với! Mình đang cần gấp. Các bạn làm được bài nào thì giúp đỡ mình nhé! Cảm ơn!

Bài 1: Cho các số thực dương a,b,c. Chứng minh rằng:

\(\frac{a^2}{\sqrt{\left(2a^2+b^2\right)\left(2a^2+c^2\right)}}+\frac{b^2}{\sqrt{\left(2b^2+c^2\right)\left(2b^2+a^2\right)}}+\frac{c^2}{\sqrt{\left(2c^2+a^2\right)\left(2c^2+b^2\right)}}\le1\).

Bài 2: Cho các số thực dương a,b,c,d. Chứng minh rằng:

\(\frac{a-b}{a+2b+c}+\frac{b-c}{b+2c+d}+\frac{c-d}{c+2d+a}+\frac{d-a}{d+2a+b}\ge0\).

Bài 3: Cho các số thực dương a,b,c. Chứng minh rằng:

\(\frac{\sqrt{b+c}}{a}+\frac{\sqrt{c+a}}{b}+\frac{\sqrt{a+b}}{c}\ge\frac{4\left(a+b+c\right)}{\sqrt{\left(a+b\right)\left(b+c\right)\left(c+a\right)}}\).

Bài 4:Cho a,b,c>0, a+b+c=3. Chứng minh rằng: 

a)\(\frac{a^3}{a^2+ab+b^2}+\frac{b^3}{b^2+bc+c^2}+\frac{c^3}{c^2+ca+a^2}\ge1\).

b)\(\frac{a^3}{a^2+b^2}+\frac{b^3}{b^2+c^2}+\frac{c^3}{c^2+a^2}\ge\frac{3}{2}\).

c)\(\frac{a+1}{b^2+1}+\frac{b+1}{c^2+1}+\frac{c+1}{a^2+1}\ge3\).

Bài 5: Cho a,b,c >0. Chứng minh rằng:

\(\frac{2a^2+ab}{\left(b+c+\sqrt{ca}\right)^2}+\frac{2b^2+bc}{\left(c+a+\sqrt{ab}\right)^2}+\frac{2c^2+ca}{\left(a+b+\sqrt{bc}\right)^2}\ge1\).

8
21 tháng 10 2019

1) Áp dụng bunhiacopxki ta được \(\sqrt{\left(2a^2+b^2\right)\left(2a^2+c^2\right)}\ge\sqrt{\left(2a^2+bc\right)^2}=2a^2+bc\), tương tự với các mẫu ta được vế trái \(\le\frac{a^2}{2a^2+bc}+\frac{b^2}{2b^2+ac}+\frac{c^2}{2c^2+ab}\le1< =>\)\(1-\frac{bc}{2a^2+bc}+1-\frac{ac}{2b^2+ac}+1-\frac{ab}{2c^2+ab}\le2< =>\)

\(\frac{bc}{2a^2+bc}+\frac{ac}{2b^2+ac}+\frac{ab}{2c^2+ab}\ge1\)<=> \(\frac{b^2c^2}{2a^2bc+b^2c^2}+\frac{a^2c^2}{2b^2ac+a^2c^2}+\frac{a^2b^2}{2c^2ab+a^2b^2}\ge1\)  (1) 

áp dụng (x2 +y2 +z2)(m2+n2+p2\(\ge\left(xm+yn+zp\right)^2\)

(2a2bc +b2c2 + 2b2ac+a2c2 + 2c2ab+a2b2). VT\(\ge\left(bc+ca+ab\right)^2\)   <=> (ab+bc+ca)2. VT \(\ge\left(ab+bc+ca\right)^2< =>VT\ge1\)  ( vậy (1) đúng)

dấu '=' khi a=b=c

21 tháng 10 2019

4b, \(\frac{a^3}{a^2+b^2}+\frac{b^3}{b^2+c^2}+\frac{c^3}{c^2+a^2}=1-\frac{ab^2}{a^2+b^2}+1-\frac{bc^2}{b^2+c^2}+1-\frac{ca^2}{a^2+c^2}\)

\(\ge3-\frac{ab^2}{2ab}-\frac{bc^2}{2bc}-\frac{ca^2}{2ac}=3-\frac{\left(a+b+c\right)}{2}=\frac{3}{2}\)

30 tháng 4 2020

\(a^2b^2c^2+\left(a+1\right)\left(1+b\right)\left(1+c\right)\ge a+b+c+ab+bc+ca+3\)

\(\Leftrightarrow\left(abc\right)^2+abc-2\ge0\Leftrightarrow\left(abc+2\right)\left(abc-1\right)\ge0\Leftrightarrow abc\ge1\)

Áp dụng BĐT Cosi ta có:

\(\frac{a^3}{\left(b+2c\right)\left(2c+3a\right)}+\frac{b+2c}{45}+\frac{2c+3a}{75}\ge3\sqrt[3]{\frac{a^3}{\left(b+2c\right)\left(2c+3b\right)}\cdot\frac{b+2c}{45}\cdot\frac{2c+3a}{75}}=\frac{a}{5}\left(1\right)\)

Tương tự ta có: \(\hept{\begin{cases}\frac{b^3}{\left(c+2a\right)\left(2a+3b\right)}+\frac{c+2a}{45}+\frac{2a+3b}{75}\ge\frac{b}{5}\left(2\right)\\\frac{c^3}{\left(a+2b\right)\left(2b+3c\right)}+\frac{a+2b}{45}+\frac{2b+3c}{75}\ge\frac{c}{5}\left(3\right)\end{cases}}\)

Từ (1)(2)(3) ta có:

\(P+\frac{2\left(a+b+c\right)}{15}\ge\frac{a+b+c}{5}\Leftrightarrow P\ge\frac{1}{15}\left(a+b+c\right)\)

Mà \(a+b+c\ge3\sqrt[3]{abc}\Rightarrow S\ge\frac{1}{5}\)

Dấu "=" xảy ra <=> a=b=c=1

3 tháng 5 2020

CHÚC BAN HỌC GIỎI